Jak możemy ograniczyć prawdopodobieństwo, że zmienna losowa jest maksymalna?


21

Załóżmy, że mamy N niezależnych zmiennych losowych X1 , , Xn ze skończonymi środkami μ1μN i wariancji σ12 , , σN2 . Szukam granic bez dystrybucji prawdopodobieństwa, że ​​każdy XiXN jest większy niż wszystkie inne Xj , ji .

Innymi słowy, jeśli dla uproszczenia założymy, że rozkłady Xi są ciągłe (takie, że P(Xi=Xj)=0 ), szukam granic na:

P(Xi=maxjXj).
Jeśli N=2 , możemy użyć nierówności Czebyszewa, aby uzyskać:
P(X1=maxjXj)=P(X1>X2)σ12+σ22σ12+σ22+(μ1μ2)2.
Chciałbym znaleźć jakieś proste (niekoniecznie mocno) Bounds do ogólnego N , ale nie byłem w stanie znaleźć (estetycznie) miłe wyników dla ogólnego N .

Należy pamiętać, że zmienne nie są uważane za id. Wszelkie sugestie lub odniesienia do powiązanych prac są mile widziane.


Aktualizacja: pamiętaj, że z założenia μjμi . Następnie możemy użyć powyższego ograniczenia, aby dojść do:

P(Xi=maxjXj)minj>iσi2+σj2σi2+σj2+(μjμi)2σi2+σN2σi2+σN2+(μNμi)2.
Oznacza to:
(μNμi)P(Xi=maxjXj)(μNμi)σi2+σN2σi2+σN2+(μNμi)212σi2+σN2.
To z kolei implikuje:
i=1NμiP(Xi=maxjXj)μNN2i=1N1(σi2+σN2).
Jestem teraz zastanawiasz się, czy ten związany można poprawić na coś, co nie zależy liniowo od N . Na przykład, czy obowiązuje:
i=1NμiP(Xi=maxjXj)μNi=1Nσi2?
A jeśli nie, to co może być kontrprzykładem?

3
To może być związany mocniej jeśli używasz indeksu , który daje im mniejsza górna granica zamiast . Zauważ, że ta wartość zależy zarówno od średniej, jak i wariancji. N.jN

5
@MichaelChernick: Nie wierzę, że to prawda. Załóżmy na przykład, że mamy trzy jednolite rozkłady na . Następnie, jeśli się nie mylę, , podczas gdy . Nie wiem, czy chciałeś napisać , ale ten sam przykład pokazuje, że nadal nie jest poprawny. p ( X 1 < max J X j ) = 2 / 3 P ( X 1 < X 2 ) = P ( x 1 < x 3 ) = 1 / 2 P ( X i > max J X j )[0,1]P(X1<maxjXj)=2/3P(X1<X2)=P(X1<X3)=1/2P(Xi>maxjXj)
MLS

2
@Michael: Niestety nadal nie jest to prawdą. Zdarzenia dla fixed nie są niezależne. Aj={Xi>Xj} i
kardynał

2
@cardinal: Między innymi dotyczy to uzbrojonych bandytów. Jeśli wybierzesz ramię w oparciu o poprzednie nagrody, jak duże jest prawdopodobieństwo, że wybrałeś najlepsze ramię (byłoby to w powyższym zapisie), i czy możemy ograniczyć spodziewaną stratę za wybranie sub -optymalne ramię? P(XN=maxjXj)
MLS

2
Przekreślony na MathOverflow: mathoverflow.net/questions/99313
kardynał

Odpowiedzi:


1

Możesz użyć nierówności Czebiszewa na wielu odmianach.

Przypadek dwóch zmiennych

W przypadku jednej sytuacji, vs. , do tej samej sytuacji, co komentarz Jochena z 4 listopada 2016 rX 2X1X2

1) Jeśli to P ( X 1 > X 2 ) ( σ 2 1 + σ 2 2 ) / ( μ 1 - μ 2 ) 2μ1<μ2P(X1>X2)(σ12+σ22)/(μ1μ2)2

(i zastanawiam się również nad twoją pochodną)

Wyprowadzenie równania 1

  • przy użyciu nowej zmiennejX1X2
  • przekształcając go tak, aby miał średnią zero
  • przyjmowanie wartości bezwzględnej
  • stosując nierówność Czebyszewa

P(X1>X2)=P(X1X2>0)=P(X1X2(μ1μ2)>(μ1μ2))P(|X1X2(μ1μ2)|>μ2μ1)σ(X1X2(μ1μ2))2(μ2μ1)2=σX12+σX22(μ2μ1)2

Przypadek wielowymiarowy

Nierówność w równaniu (1) można zmienić na przypadek wielowymiarowy, stosując ją do wielu transformowanych zmiennych dla każdego (zauważ, że są one skorelowane).i < n(XnXi)i<n

Rozwiązanie tego problemu (wielowymiarowe i skorelowane) zostało opisane przez I. Olkina i JW Pratta. „A Multivariate Tchebycheff Inequality” w Annals of Mathematical Statistics, tom 29 strony 226-234 http://projecteuclid.org/euclid.aoms/1177706720

Uwaga twierdzenie 2.3

P(|yi|kiσi for some i)=P(|xi|1 for some i)(u+(ptu)(p1))2p2

w którym liczba zmiennych, , a .t = k - 2 i u = ρ i j / ( k i k j )pt=ki2u=ρij/(kikj)

Twierdzenie 3.6 zapewnia ściślejsze powiązanie, ale jest trudniejsze do obliczenia.

Edytować

Ostrzejsze granice można znaleźć, stosując nierówność Cantelliego na wielu odmianach . Ta nierówność jest typem, który użyłeś wcześniej i zapewnił ci granicę który jest ostrzejsze niż .( σ 2 1 + σ 2 2 ) / ( μ 1 - μ 2 ) 2(σ12+σ22)/(σ12+σ22+(μ1μ2)2)(σ12+σ22)/(μ1μ2)2

Nie poświęciłem czasu na przestudiowanie całego artykułu, ale i tak możesz znaleźć rozwiązanie tutaj:

AW Marshall i I. Olkin „Jednostronna nierówność typu Czebyszewa” w Annals of Mathematical Statistics tom 31 str. 488-491 https://projecteuclid.org/euclid.aoms/1177705913

(późniejsza uwaga: ta nierówność dotyczy równych korelacji i niewystarczającej pomocy. W każdym razie twój problem, aby znaleźć najostrzejszą granicę, jest równy, bardziej ogólnie, wielowymiarowej nierówności Cantellego. Byłbym zaskoczony, gdyby rozwiązanie nie istniało)


Czy możesz podać wyraźne stwierdzenie wielowymiarowej nierówności Czebyszewa?
whuber

1
Zredagowałem rozwiązanie podając całe twierdzenie.
Sextus Empiricus

-1

Znalazłem twierdzenie, które może ci pomóc i spróbuję je dostosować do twoich potrzeb. Załóżmy, że masz:

exp(tE(max1inXi))

Następnie przez nierówność Jensena (ponieważ exp (.) Jest funkcją wypukłą), otrzymujemy:

exp(tE(max1inXi))E(exp(tmax1inXi))=E(max1in exp(tXi))i=1nE(exp(tXi)

Teraz w przypadku musisz podłączyć dowolną funkcję generującą moment swojej zmiennej losowej (ponieważ jest to tylko definicja mgf). Następnie, po zrobieniu tego (i potencjalnie upraszczając twój termin), bierzesz ten termin, bierzesz dziennik i dzielisz go przez t, aby uzyskać oświadczenie o terminie . Następnie możesz wybrać t z dowolną wartością (najlepiej, aby termin był mały, aby granica była ciasna). X i E ( m a x 1 i n X i )exp(tXiXiE(max1inXi)

Następnie masz oświadczenie o oczekiwanej wartości maksymalnej ponad n rvs. Aby uzyskać teraz stwierdzenie o prawdopodobieństwie, że maksimum tych wartości rv odbiega od tej oczekiwanej wartości, możesz po prostu użyć nierówności Markowa (zakładając, że twoje wartości rv jest nieujemne) lub innej, bardziej szczegółowej wartości rv, odnoszącej się do twojego konkretnego wartości.

Korzystając z naszej strony potwierdzasz, że przeczytałeś(-aś) i rozumiesz nasze zasady używania plików cookie i zasady ochrony prywatności.
Licensed under cc by-sa 3.0 with attribution required.